Proszę o wytłumaczenie.

Własności ciągów i zbieżność, obliczanie granic. Twierdzenia o zbieżności.
Dynia5
Użytkownik
Użytkownik
Posty: 43
Rejestracja: 28 maja 2023, o 15:40
Płeć: Mężczyzna
wiek: 19
Podziękował: 4 razy
Pomógł: 2 razy

Proszę o wytłumaczenie.

Post autor: Dynia5 »

Mam pytanie dlaczego ten ciąg jest równy właśnie tyle skąd to wynika?
\(\displaystyle{ \frac{1}{n}+\frac{1}{n+1}+...+\frac{1}{2n}= \frac{n+1}{2n} }\)
Jan Kraszewski
Administrator
Administrator
Posty: 34123
Rejestracja: 20 mar 2006, o 21:54
Płeć: Mężczyzna
Lokalizacja: Wrocław
Podziękował: 3 razy
Pomógł: 5192 razy

Re: Proszę o wytłumaczenie.

Post autor: Jan Kraszewski »

To nieprawda. Sprawdź dla \(\displaystyle{ n=1.}\)

JK
Dynia5
Użytkownik
Użytkownik
Posty: 43
Rejestracja: 28 maja 2023, o 15:40
Płeć: Mężczyzna
wiek: 19
Podziękował: 4 razy
Pomógł: 2 razy

Re: Proszę o wytłumaczenie.

Post autor: Dynia5 »

A dla \(\displaystyle{ n\geq 2}\) to prawda?
Jan Kraszewski
Administrator
Administrator
Posty: 34123
Rejestracja: 20 mar 2006, o 21:54
Płeć: Mężczyzna
Lokalizacja: Wrocław
Podziękował: 3 razy
Pomógł: 5192 razy

Re: Proszę o wytłumaczenie.

Post autor: Jan Kraszewski »

A czemu sobie nie sprawdzisz np. dla \(\displaystyle{ n=2}\) ?

Może do tej równości jest dodane jakieś polecenie?

JK
Dynia5
Użytkownik
Użytkownik
Posty: 43
Rejestracja: 28 maja 2023, o 15:40
Płeć: Mężczyzna
wiek: 19
Podziękował: 4 razy
Pomógł: 2 razy

Re: Proszę o wytłumaczenie.

Post autor: Dynia5 »

Jest polecenie:
Udowodnij prawdziwość nierówności stosując zasadę indukcji:
\(\displaystyle{ \frac{1}{n}+\frac{1}{n+1}+...+\frac{1}{2n}>\frac{1}{2}}\) dla \(\displaystyle{ n \in \NN_+}\). Jak to zrobić?
Ostatnio zmieniony 28 maja 2023, o 17:10 przez Jan Kraszewski, łącznie zmieniany 1 raz.
Powód: Poprawa wiadomości.
Jan Kraszewski
Administrator
Administrator
Posty: 34123
Rejestracja: 20 mar 2006, o 21:54
Płeć: Mężczyzna
Lokalizacja: Wrocław
Podziękował: 3 razy
Pomógł: 5192 razy

Re: Proszę o wytłumaczenie.

Post autor: Jan Kraszewski »

No to pokaż, jak robisz tę indukcję.

JK
Dynia5
Użytkownik
Użytkownik
Posty: 43
Rejestracja: 28 maja 2023, o 15:40
Płeć: Mężczyzna
wiek: 19
Podziękował: 4 razy
Pomógł: 2 razy

Re: Proszę o wytłumaczenie.

Post autor: Dynia5 »

Umieściłem to zadanie na forum zadnia.info tutaj adres- https://forum.zadania.info/viewforum.php?f=32 . W pierwszym rozwiązaniu autor na końcu przy porównaniu zapisał to wten sposób i nie wiem dlaczego tak jest.
Jan Kraszewski
Administrator
Administrator
Posty: 34123
Rejestracja: 20 mar 2006, o 21:54
Płeć: Mężczyzna
Lokalizacja: Wrocław
Podziękował: 3 razy
Pomógł: 5192 razy

Re: Proszę o wytłumaczenie.

Post autor: Jan Kraszewski »

Nie komentuję rozwiązań z innego forum. Jak chcesz odpowiedzi, to przepisz rozumowanie tutaj. A najlepiej zrób je sam...

JK
Dynia5
Użytkownik
Użytkownik
Posty: 43
Rejestracja: 28 maja 2023, o 15:40
Płeć: Mężczyzna
wiek: 19
Podziękował: 4 razy
Pomógł: 2 razy

Re: Proszę o wytłumaczenie.

Post autor: Dynia5 »

\( \frac{1}{n}+\frac{1}{n+1}+...+\frac{1}{2n}-\frac{1}{2}>0\)
Powinniśmy wywnioskować
\( \frac{1}{n+1}+\frac{1}{n+2}+...+\frac{1}{2n+2}-\frac{1}{2}>0\)
i standardowo w dowodach indukcyjnych:
\(L_N=\color{green}{\frac{1}{n}}+\frac{1}{n+1}+\frac{1}{n+2}+...+\frac{1}{2n}+\color{blue}{\frac{1}{2n+1}+\frac{1}{2n+2}}-\frac{1}{2}\color{green}{-\frac{1}{n}}{>}0+\color{blue}{\frac{1}{2n+1}+\frac{1}{2n+2}}\color{green}{-\frac{1}{n}}=\frac{-3n-2}{(2n+1)(2n+2)n}\)
ale to szacowanie jest "za grube"... dla tej prawdziwej nierówności, bo
\(\underbrace{\frac{1}{n}+\frac{1}{n+1}+...+\frac{1}{2n}}_{(n+1)\text{ składników}}>\underbrace{\frac{1}{2n}+\frac{1}{2n}+...+\frac{1}{2n}}_{(n+1)\text{ składników}}=\frac{n+1}{2n}>\frac{n}{2n}={1\over2}\) interesuje mnie fragment zaznaczony na czerwono dlaczego jest tutaj \(\displaystyle{ n+1}\) składników i dlaczego to wyrażenie o którym pisałem jest tutaj właśnie tyle równe.
Jan Kraszewski
Administrator
Administrator
Posty: 34123
Rejestracja: 20 mar 2006, o 21:54
Płeć: Mężczyzna
Lokalizacja: Wrocław
Podziękował: 3 razy
Pomógł: 5192 razy

Re: Proszę o wytłumaczenie.

Post autor: Jan Kraszewski »

Dynia5 pisze: 28 maja 2023, o 17:53ale to szacowanie jest "za grube"... dla tej prawdziwej nierówności, bo
\(\underbrace{\frac{1}{n}+\frac{1}{n+1}+...+\frac{1}{2n}}_{(n+1)\text{ składników}}>\underbrace{\frac{1}{2n}+\frac{1}{2n}+...+\frac{1}{2n}}_{(n+1)\text{ składników}}=\frac{n+1}{2n}>\frac{n}{2n}={1\over2}\) interesuje mnie fragment zaznaczony na czerwono dlaczego jest tutaj \(\displaystyle{ n+1}\) składników i dlaczego to wyrażenie o którym pisałem jest tutaj właśnie tyle równe.
Masz

\(\displaystyle{ \frac{1}{n}+\frac{1}{n+1}+...+\frac{1}{2n}=\frac{1}{n\blue{+0}}+\frac{1}{n\blue{+1}}+...+\frac{1}{n\blue{+n}}}\)

czyli składników jest \(\displaystyle{ n+1}\) (bo tyle jest liczb od \(\displaystyle{ 0}\) do \(\displaystyle{ n}\)).

Potem masz szacowanie z dołu: każdy z tych \(\displaystyle{ n+1}\) wyrazów szacujesz przez najmniejszy z nich, czyli \(\displaystyle{ \frac{1}{2n}.}\) A potem sumujesz \(\displaystyle{ n+1}\) identycznych liczb \(\displaystyle{ \frac{1}{2n},}\) więc dostajesz \(\displaystyle{ \frac{n+1}{2n}.}\) W swoim wyjściowym poście z jakiegoś powodu zastąpiłeś nierówność równością i dlatego dostałeś nieprawdę.

JK
Awatar użytkownika
Dasio11
Moderator
Moderator
Posty: 10211
Rejestracja: 21 kwie 2009, o 19:04
Płeć: Mężczyzna
Lokalizacja: Wrocław
Podziękował: 40 razy
Pomógł: 2359 razy

Re: Proszę o wytłumaczenie.

Post autor: Dasio11 »

Dynia5 pisze: 28 maja 2023, o 17:53\( \frac{1}{n}+\frac{1}{n+1}+...+\frac{1}{2n}-\frac{1}{2}>0\)
Powinniśmy wywnioskować
\( \frac{1}{n+1}+\frac{1}{n+2}+...+\frac{1}{2n+2}-\frac{1}{2}>0\)
i standardowo w dowodach indukcyjnych:
\(L_N=\color{green}{\frac{1}{n}}+\frac{1}{n+1}+\frac{1}{n+2}+...+\frac{1}{2n}+\color{blue}{\frac{1}{2n+1}+\frac{1}{2n+2}}-\frac{1}{2}\color{green}{-\frac{1}{n}}{>}0+\color{blue}{\frac{1}{2n+1}+\frac{1}{2n+2}}\color{green}{-\frac{1}{n}}=\frac{-3n-2}{(2n+1)(2n+2)n}\)
Dynia5 pisze: 28 maja 2023, o 17:53\(\underbrace{\frac{1}{n}+\frac{1}{n+1}+...+\frac{1}{2n}}_{(n+1)\text{ składników}}>\underbrace{\frac{1}{2n}+\frac{1}{2n}+...+\frac{1}{2n}}_{(n+1)\text{ składników}}=\frac{n+1}{2n}>\frac{n}{2n}={1\over2}\)
Dodam, że powyższe dwa fragmenty nie mają ze sobą związku. Pierwszy ilustruje, dlaczego najbardziej bezpośrednia próba dowodu indukcyjnego jest nieskuteczna. Drugi zaś pokazuje (bez indukcji), że problem jest w metodzie a nie w twierdzeniu, bo ono (tj. nierówność) jest prawdziwe.

Jeśli chcesz udowodnić nierówność indukcyjnie, to skorzystaj z rady Icanseepeace z 13:06.
Dynia5
Użytkownik
Użytkownik
Posty: 43
Rejestracja: 28 maja 2023, o 15:40
Płeć: Mężczyzna
wiek: 19
Podziękował: 4 razy
Pomógł: 2 razy

Re: Proszę o wytłumaczenie.

Post autor: Dynia5 »

A potem sumujesz \(\displaystyle{ n+1}\) identycznych liczb \(\displaystyle{ \frac{1}{2n},}\)
A jak wszystkie liczby tutaj są równe \(\displaystyle{ \frac{1}{2n} }\)?
Jan Kraszewski
Administrator
Administrator
Posty: 34123
Rejestracja: 20 mar 2006, o 21:54
Płeć: Mężczyzna
Lokalizacja: Wrocław
Podziękował: 3 razy
Pomógł: 5192 razy

Re: Proszę o wytłumaczenie.

Post autor: Jan Kraszewski »

Znasz sformułowanie "oszacowanie z dołu"? Wiesz, że:

\(\displaystyle{ \frac{1}{n}>\frac{1}{2n}\\
\frac{1}{n+1}>\frac{1}{2n}\\
\frac{1}{n+2}>\frac{1}{2n}\\
...\\
\frac{1}{2n-1}>\frac{1}{2n}\\
\frac{1}{2n}\ge\frac{1}{2n}
}\)


Jak dodasz te nierówności stronami, to dostaniesz

\(\displaystyle{ \frac{1}{n}+\frac{1}{n+1}+...+\frac{1}{2n}>\frac{1}{2n}+\frac{1}{2n}+...+\frac{1}{2n}.}\)

JK
Dynia5
Użytkownik
Użytkownik
Posty: 43
Rejestracja: 28 maja 2023, o 15:40
Płeć: Mężczyzna
wiek: 19
Podziękował: 4 razy
Pomógł: 2 razy

Re: Proszę o wytłumaczenie.

Post autor: Dynia5 »

A tutaj nie otrzyłameś \(\displaystyle{ \frac{n+1}{2n} }\) o które mnie chodzi
a4karo
Użytkownik
Użytkownik
Posty: 22171
Rejestracja: 15 maja 2011, o 20:55
Płeć: Mężczyzna
Lokalizacja: Bydgoszcz
Podziękował: 38 razy
Pomógł: 3748 razy

Re: Proszę o wytłumaczenie.

Post autor: a4karo »

Sam już nic nie chcesz policzyć?
ODPOWIEDZ